Why is D incorrect?
I originally picked D but on review picked B; however, what is the correct reasoning that D is no...
Meredith on December 20 at 07:53PM
  • December 2004 LSAT
  • SEC3
  • Q7
1
Reply
Choice C
Why is C wrong?
Eliyahu-Ohevshalom on December 20 at 07:51PM
  • December 2004 LSAT
  • SEC3
  • Q7
1
Reply
Answer D
Hi LSAT Max! I was wondering if someone could explain how we can get to D as the correct answ...
Julie-V on December 20 at 07:44PM
  • December 2004 LSAT
  • SEC3
  • Q3
1
Reply
What about E?
It looked like there were indications on the passage that the need for short-term aid is also cru...
SoYoung on December 20 at 07:42PM
  • December 2004 LSAT
  • SEC3
  • Q3
1
Reply
Why wouldn't it be answer A?
I didn't find anything in the passage that would say the reformers would believe this to be wrong...
Annabelle20115 on August 7 at 04:06PM
  • December 2004 LSAT
  • SEC3
  • Q9
1
Reply
E...
How are we supposed to have prior knowledge about facelifts versus a major surgery to understand ...
sia.aleem9@gmail.com on November 28, 2022
  • December 2004 LSAT
  • SEC3
  • Q14
1
Reply
Differentiate why C is better than E
I know I fell into the trap of the final sentence = main point, instead of looking back at the fu...
rswafra on April 27, 2022
  • December 2004 LSAT
  • SEC3
  • Q20
1
Reply
Why is C wrong?
I am unsure as to why C is wrong on this question. Could I please get some input on why this is t...
camerongreen13 on April 19, 2022
  • December 2004 LSAT
  • SEC3
  • Q10
2
Replies
Answer Choice E Meaning
Just a bit of confusion as to what the answer choice is saying, especially the last part: "Th...
JYL on March 6, 2022
  • December 2004 LSAT
  • SEC3
  • Q26
1
Reply
Can you please explain why E is wrong?
Hello LSATMax team. Could someone please explain to me why E is incorrect? I interpreted the last...
Juan.23 on January 16, 2022
  • December 2004 LSAT
  • SEC3
  • Q10
2
Replies
Between B and C
I was stuck between both B and C. While I initially was going to choose B, after a second thought...
BaileyPyne on April 16, 2021
  • December 2004 LSAT
  • SEC3
  • Q7
2
Replies
Error in Answer B?
Is answer choice (b) wrong because it states "provincial courts are not authorized to rule on the...
Mansi-Arora on January 7, 2021
  • December 2004 LSAT
  • SEC3
  • Q26
1
Reply
Question
Why is A incorrect
Meckena-Hultin on September 21, 2020
  • December 2004 LSAT
  • SEC3
  • Q8
3
Replies
Answer Explanations
Hello! I was wondering is someone could explain why D is the correct answer and how we can el...
Julie-V on September 10, 2020
  • December 2004 LSAT
  • SEC3
  • Q15
3
Replies
Why is A the correct answer?
I would like an explanation of why A is correct, please
naieka99 on August 11, 2020
  • December 2004 LSAT
  • SEC3
  • Q6
1
Reply
Answer D
Why is D wrong? I can understand why the right answer is what it is, but I can’t see why I should...
izzy on July 15, 2020
  • December 2004 LSAT
  • SEC3
  • Q2
3
Replies
Answer A
Like the two other unanswered individuals, I’m having trouble understanding why A is incorrect. I...
izzy on July 14, 2020
  • December 2004 LSAT
  • SEC3
  • Q7
1
Reply
Answer E
I don’t understand why E is correct
izzy on July 14, 2020
  • December 2004 LSAT
  • SEC3
  • Q14
1
Reply
Correct Answer
What is the correct answer here? I am thinking C, but I'm not finding anywhere where the correct/...
James-Nash on June 25, 2020
  • December 2004 LSAT
  • SEC3
  • Q20
1
Reply
Choice D
What makes D incorrect?
bcross on June 18, 2020
  • December 2004 LSAT
  • SEC3
  • Q2
2
Replies